How to prove an interval is open?

  • Thread starter Thread starter shredder666
  • Start date Start date
  • Tags Tags
    Interval
Click For Summary
An open interval, such as (1,2), is defined as open because for any point within the interval, there exist additional points that can be found arbitrarily close to the boundaries without including them. This means that for any point x in (1,2), you can always find points closer to 1 and 2, demonstrating that the interval does not include its endpoints. In contrast, a closed interval like [1,2] includes its endpoints, meaning there are no additional points beyond 2. The concept of neighborhoods relates to this idea, where a neighborhood around a point allows for the existence of other points within the interval. Understanding this distinction clarifies why open intervals are characterized as such.
shredder666
Messages
63
Reaction score
0

Homework Statement


I can repeat the proof, but then I won't understand it.
How do I prove that an open interval like (1,2) is open?

Homework Equations


I know it has something to do with neighbourhood with a given epsilon, but I don't really get what a neighbourhood is, I have a maybe rough picture of it but not sufficient enough
it'd be nice if you could explain in some sort of analogy or plain english :)

The Attempt at a Solution


let x be in (1,2)
|x-1|>=e |x-2|=>e
e=min{x-5,17-x}
then neighbouthood will be inside (1,2)
and I just get lost from there
 
Physics news on Phys.org
Here's some plain English:

A set is open if for any point inside the set, you can always find more points between here and the edge. So (1,2) is open because if you are at 1.99, you can find 1.999, and so on. But [1,2] is closed because if you are at 2, then there are no more points between here and the edge.
 
Question: A clock's minute hand has length 4 and its hour hand has length 3. What is the distance between the tips at the moment when it is increasing most rapidly?(Putnam Exam Question) Answer: Making assumption that both the hands moves at constant angular velocities, the answer is ## \sqrt{7} .## But don't you think this assumption is somewhat doubtful and wrong?

Similar threads

  • · Replies 4 ·
Replies
4
Views
2K
  • · Replies 1 ·
Replies
1
Views
2K
  • · Replies 3 ·
Replies
3
Views
5K
  • · Replies 4 ·
Replies
4
Views
2K
  • · Replies 12 ·
Replies
12
Views
2K
Replies
10
Views
1K
  • · Replies 17 ·
Replies
17
Views
2K
Replies
32
Views
3K
  • · Replies 26 ·
Replies
26
Views
3K
Replies
1
Views
1K